Đến nội dung

MrS nội dung

Có 33 mục bởi MrS (Tìm giới hạn từ 23-05-2020)



Sắp theo                Sắp xếp  

#646537 Gõ thử công thức toán

Đã gửi bởi MrS on 26-07-2016 - 11:00 trong Thử các chức năng của diễn đàn

$$Cho a,b,c không âm thỏa mãn a + b + c = 1, Tìm giá trị lớn nhất: P = a^{2}b+b^{2}c+c^{2}a$ Giả sử$




#646538 Gõ thử công thức toán

Đã gửi bởi MrS on 26-07-2016 - 11:03 trong Thử các chức năng của diễn đàn

Cho a, b ,c là các số thực không âm thỏa mãn a + b + c = 1,

Tìm GTLN của P = $$a(b-c)(b-a)\leq 0\Leftrightarrow c(b^{2}-ab-bc+ca)\Leftrightarrow b^{2}c+c^{2}a\leq c^{2}b+abc$ 

 

Lời giải:

Không mất tính tổng quát giải sử b là số nằm giữa a và c, khi đó:

$a(b-c)(b-a)\leq 0\Leftrightarrow a(b^{2}-ab-bc+ca)\Leftrightarrow ab^{2}+ca^{2}\leq a^{2}b+abc$

Suy ra $P=$a^{2}b+b^{2}c+c^{2}a$\leq c^{2}b+abc+a^{2}b=b(c^{2}+ca+c^{2})\leq b(c+a)^{2}$$=16\cdot \frac{b}{2}\cdot \frac{b}{2}\cdot \frac{a+c}{2}\cdot \frac{a+c}{2}\leq 16\cdot \left ( \frac{\frac{b}{2}+\frac{b}{2}+\frac{a+c}{2}+\frac{a+c}{2}}{3} \right )^{3}=16\cdot \left ( \frac{a+b+c}{3} \right )^{3}=\frac{16}{9}$

 

Vậy MaxP = \frac{16}{9}. Đạt được khi $a=b=\frac{1}{2}, c=0$ hoặc các hoán vị 




#646575 Gõ thử công thức toán

Đã gửi bởi MrS on 26-07-2016 - 16:32 trong Thử các chức năng của diễn đàn

Cho a, b ,c là các số không âm thỏa mãn (a+b)c>0

Tìm MinP vơi $P = \sqrt {\frac{a}{{b + c}}} + \sqrt {\frac{b}{{a + c}}} + \frac{c}{{2\left( {a + b} \right)}}$

Theo AM - GM: 

$\sqrt{a(b+c)}\leq \frac{a+b+c}{2} \Rightarrow \sqrt {\frac{a}{{b + c}}}\cdot \sqrt{a(b+c)}\leq \frac{a+b+c}{2}\cdot \sqrt {\frac{a}{{b + c}}}\Rightarrow \sqrt {\frac{a}{{b + c}}}\geq \frac{2a}{a+b+c}$. Đẳng thức xảy ra khi a=0 hoặc a=b+c.

Tương tự: $\sqrt {\frac{b}{{c + a}}}\geq \frac{2b}{a+b+c}$. Đẳng thức xảy ra khi b=0 hoặc b=a+c

$VT\geq \frac{2(a+b))}{a+b+c}+\frac{c}{2(a+b)}=\frac{2}{1+\frac{c}{a+b}}+\frac{c}{2(a+b)}=\frac{2}{1+t}+\frac{t}{2}=\frac{2}{1+t}+\frac{t+1}{2}-\frac{1}{2}\geq 2-\frac{1}{2}=\frac{3}{2}$. Đẳng thức xảy ra khi c=a+b

Dấu bằng xảy ra khi: a=0, b=c nhoặc b=0, c=a.




#646576 [TSĐH 2014] Đề thi khối B

Đã gửi bởi MrS on 26-07-2016 - 16:51 trong Thi TS ĐH

Theo AM - GM: 

$\sqrt{a(b+c)}\leq \frac{a+b+c}{2} \Rightarrow \sqrt {\frac{a}{{b + c}}}\cdot \sqrt{a(b+c)}\leq \frac{a+b+c}{2}\cdot \sqrt {\frac{a}{{b + c}}}\Rightarrow \sqrt {\frac{a}{{b + c}}}\geq \frac{2a}{a+b+c}$. Đẳng thức xảy ra khi a=0 hoặc a=b+c.

Tương tự: $\sqrt {\frac{b}{{c + a}}}\geq \frac{2b}{a+b+c}$. Đẳng thức xảy ra khi b=0 hoặc b=a+c

$VT\geq \frac{2(a+b))}{a+b+c}+\frac{c}{2(a+b)}=\frac{2}{1+\frac{c}{a+b}}+\frac{c}{2(a+b)}=\frac{2}{1+t}+\frac{t}{2}=\frac{2}{1+t}+\frac{t+1}{2}-\frac{1}{2}\geq 2-\frac{1}{2}=\frac{3}{2}$. Đẳng thức xảy ra khi c=a+b

Dấu bằng xảy ra khi: a=0, b=c hoặc b=0, c=a.




#646585 Gõ thử công thức toán

Đã gửi bởi MrS on 26-07-2016 - 17:32 trong Thử các chức năng của diễn đàn

Cho a, b, c >0 thỏa $(a+c)(b+c)=4c^2. Tìm GTNN của:

$P=\frac{32a^3}{(b+3c)^3}+\frac{32b^3}{(a+3c)^3}-\frac{\sqrt{a^2+b^2}}{c}$= $=\frac{32x^{3}}{(y+3)^{3}}+\frac{32y^{3}}{(x+3)^{3}}-\sqrt{x^2+y^2}$ thỏa mãn $(x+1)(y+1)=4$  

Theo AM = GM: $\frac{32x^{3}}{(y+3)^{3}}+4+4\geq 3\sqrt[3]{4\cdot 4\cdot \frac{32x^{3}}{(y+3)^{3}}}=24\cdot \frac{x}{y+3}$

Tương tự: $\frac{32y^{3}}{(x+3)^{3}}+4+4 \geq 24\cdot \frac{y}{x+3}$$

Đặt x+1=m, y+1=n.Suy ra mn=4 (m,n >1)

$VT\geq \frac{24x}{y+3}+\frac{24y}{x+3}-\sqrt{x^2+y^2}=\frac{24((m-1)(n-1+3)+(n-1)(m-1+3)))}{(m-1+3)(n-1+3)}-\sqrt{(m-1)^2+(n-1)^2}=\frac{24((m-1)(n+2)+(n-1)(m+2)))}{(m+2)(n+2)}+\sqrt{m^2+n^2-2(m+n)+2}=\frac{24(2mn+2m-n-2+2n-m-2)}{mn+2(m+n)+4}+\sqrt{(m+n)^2-2mn-2(m+n)+2}=\frac{24(2mn+m+n-4)}{mn+2(m+n)+4}+\sqrt{(m+n)^2-2(m+n)-6}=\frac{24(m+n)}{2(m+n)+8}-\sqrt{(m+n)^2-2(m+n)-6}$

Đặt $m+n=t$, vì mn=4 nên $4=mn\leq (\frac{m+n}{2})^2=\frac{t^2}{4}\Rightarrow t\geq 4$

Xét hàm số: 




#646680 Gõ thử công thức toán

Đã gửi bởi MrS on 27-07-2016 - 10:49 trong Thử các chức năng của diễn đàn

Bạn chỉ thêm kí tự $   $ vào trước và sau công thức toán thôi, không phải cả câu đâu

Lần đầu em nghịch cái này bác à  :icon6:




#650235 Topic: [LTDH] Mỗi ngày hai bất đẳng thức.

Đã gửi bởi MrS on 18-08-2016 - 16:20 trong Bất đẳng thức và cực trị

Và tiếp theo là hai bài sau:

Bài 3: Cho $a,b,c$ là các số thực dương thỏa mãn: $a+b+c=3$. Chứng minh rằng: $3(a^2+b^2+c^2)+4abc\ge 13$.

Bài 4: Cho $x,y,z>1$ thỏa mãn: $x+y+z=xyz$. Tìm min: $X=\frac{x-2}{z^2}+\frac{y-2}{x^2}+\frac{z-2}{y^2}$.

Bài 3: Giải bằng dồn biến: $Giả sử a=min(a,b,c)\Rightarrow a\leq 1$

Đặt $f(a,b,c)=3(a^2+b^2+c^2)+4abc-13$

Đặt t=(b+c)/2 xét $f(a,b,c)- f(a,t,t)= (3/2-a)(b-c)^2\geq 0$ do $a\leq 1$ 

$f(a,t,t)=3(a^2+2t^2)+4at^2-13$, ta có a+2t=3 nên a=3-2t, thế vào và rút gọn ta được: $2(t-1)^2(7-4t)\geq 0$ do $t=\frac{b+c}{2}<\frac{a+b+c}{2}=\frac{3}{2}$ nên 7-4t>0

BĐT được chứng minh. Đẳng thức xảy ra khi a=b=c=1 




#652501 Topic: [LTDH] Mỗi ngày hai bất đẳng thức.

Đã gửi bởi MrS on 02-09-2016 - 23:08 trong Bất đẳng thức và cực trị

Giúp em nốt bài này nhé! Cho a,b,c>0,abc=1,tìm max của \sum \frac{a}{a+b^2+c^2}

Ta sẽ chứng minh $\sum \frac{a}{a+b^2+c^2}\leq 1\Leftrightarrow P=\sum \frac{b^2+c^2}{a+b^2+c^2}\geq 2$

Chú ý theo BĐT Cauchy - Schwarz thì: $b^2+c^2\geq \frac{(b+c)^2}{2}$ nên $P=\sum \frac{b^2+c^2}{a+b^2+c^2}\geq \sum \frac{\frac{(b+c)^2}{2}}{a+\frac{(b+c)^2}{2}}=\sum \frac{(b+c)^2}{2a+(b+c)^2}\geq \frac{4(a+b+c)^2)}{2\sum a+\sum (b+c)^2}=A$

Ta sẽ chứng minh $A\geq 2$ hay $2(a+b+c)^2\geq 2\sum a+\sum (b+c)^2\Leftrightarrow ab+bc+ca\geq a+b+c\Leftrightarrow f(a,b,c)=ab+bc+ca-a-b-c\geq 0$

Xét $d=f(a,b,c)-f(a,\sqrt{bc},\sqrt{bc})=(\sqrt{b}-\sqrt{c})^2(a-1)$

Không mất tổng quát giả sử $a=max(a,b,c)$ khi đó $a\geq 1$ suy ra $d\geq 0$

Vậy ta chỉ cần chứng minh $f(a,t,t)=t^2+2at-a-2t\geq 0$ với $t=\sqrt{bc}$  từ điều kiện $at^2=1\Rightarrow a=\frac{1}{t^2}$ thế vào rút gọn ta được $f(a,t,t)=\frac{(t-1)^2(t+1)}{t^2}\geq 0$ với mọi $t> 0$

Bắt đẳng thức được chứng minh

Vậy $MaxP=1$ tại $a=b=c=1$.




#654866 CMR: $\sum \frac{(b+c-a)^2}{(b+c)^2+a^2}...

Đã gửi bởi MrS on 20-09-2016 - 13:49 trong Bất đẳng thức và cực trị

 

Cho $a,\,b,\,c$ là ba số thực sao cho $a^2 + b^2 + c^2 > 0.$ Chứng minh rằng

$$\frac{(b+c-a)^2}{(b+c)^2+a^2}+\frac{(c+a-b)^2}{(c+a)^2+b^2}+\frac{(a+b-c)^2}{(a+b)^2+c^2}\geq \frac{3}{5}.$$

 

Chuẩn hóa a+b+c =3, BĐT tương đương với:

$\sum \frac{1}{2a^2-6a+9}\leq \frac{3}{5}$

Dễ thấy: $\frac{1}{2x^2-6x+9}\leq \frac{2x+3}{25}$ với mọi $x\geq \frac{-1}{2}$ (nó tương đương với: $\frac{2(x-1)^2(2x+1)}{2x^2-6x+9}\geq 0$, mọi $x\geq \frac{-1}{2}$ )

Giả sử: $a\geq b\geq c\Rightarrow a\geq 1\geq c$

TH1: Nếu $a\geq b\geq c\geq \frac{-1}{2}$ thì $VT\leq \frac{2\sum a+9}{25}=\frac{3}{5}$

TH2: Nếu $c< \frac{-1}{2}$, dễ thấy $f(c)=\frac{1}{2c^2-6c+9}$ đồng biến với  $c< \frac{-1}{2}$ nên $f(c)< f(\frac{-1}{2})=\frac{2}{25}$

Lại có $\frac{1}{2a^2-6a+9}+\frac{1}{2b^2-6b+9}=\frac{1}{2(a-3/2)^2+9/2}+\frac{1}{2(b-3/2)^2+9/2}< \frac{1}{9/2}+\frac{1}{9/2}=\frac{4}{9}$

Từ đó suy ra $VT< \frac{2}{25}+\frac{4}{9}=\frac{118}{225}<\frac{3}{5}$

Từ các TH suy ra Đpcm. 




#654896 Topic: [LTDH] Mỗi ngày hai bất đẳng thức.

Đã gửi bởi MrS on 20-09-2016 - 18:26 trong Bất đẳng thức và cực trị

Tiếp theo:

Bài 47: Cho $a,b,c$ là các số thực dương thỏa mãn: $(3a+2b+c)(\frac{1}{a}+\frac{2}{b}+\frac{3}{c})=30$. Tìm giá trị lớn nhất của biểu thức:

$P=\frac{b+2c-7\sqrt{72a^2+c^2}}{a}$.

Bài 48: Cho $a,b,c$ là các số thực dương. Tìm giá trị nhỏ nhất của:

$P=\sqrt{\frac{a}{b+c}}+\sqrt{\frac{b}{c+a}}+\sqrt{\frac{c}{b+a}}+\frac{\sqrt{2}(a+b+c)}{\sqrt{ab}+\sqrt{bc}+\sqrt{ca}}$.

Bài 48:

Không mất tổng quát giả sử c=min(a, b ,c).

Trước tiên ta CM bổ đề quen thuộc: $\sqrt{\frac{a}{b+c}}+\sqrt{\frac{b}{c+a}}\geq 2\sqrt{\frac{a+b}{a+b+2c}}$ ( có thể dùng Holder để chứng minh)

Lại theo AM - GM: $\sqrt{ab}+\sqrt{bc}+\sqrt{ca}\leq \frac{a+b}{2}+\sqrt{2c(a+b)}$

Đặt: $\sqrt{\frac{2c}{b+a}}=t$ với t thuộc (0;1]

Bài toán đưa về tìm Min của: $f(t)=\frac{2}{\sqrt{t^2+1}}+\frac{t}{\sqrt{2}}+\frac{\sqrt{2}(t^2+1)}{2t+1}$

Ta có: $f'(t)=-2t(t^2+1)^{-3/2}+\frac{1}{\sqrt{2}}+2\sqrt{2}(t^2+t-2)(2t+1)^{-2}$ và $f''(t)=2(2t^2-1)(t^2+1)^{-5/2}+18\sqrt{2}(2t+3)^{-3}$

Dễ thấy rằng $f"(t)>0$ với mọi với t thuộc (0;1]

Do đó f'(t) có duy nhất nghiệm t=1.

Khảo sát ta được $Minf(t)=f(1)=\frac{5\sqrt{2}}{2}$

Vậy Min P = $\frac{5\sqrt{2}}{2}$. Đẳng thức xảy ra khi a=b=c




#654897 Topic: [LTDH] Mỗi ngày hai bất đẳng thức.

Đã gửi bởi MrS on 20-09-2016 - 18:28 trong Bất đẳng thức và cực trị

Bài 48:

Không mất tổng quát giả sử c=min(a, b ,c).

Trước tiên ta CM bổ đề quen thuộc: $\sqrt{\frac{a}{b+c}}+\sqrt{\frac{b}{c+a}}\geq 2\sqrt{\frac{a+b}{a+b+2c}}$ ( có thể dùng Holder để chứng minh)

Lại theo AM - GM: $\sqrt{ab}+\sqrt{bc}+\sqrt{ca}\leq \frac{a+b}{2}+\sqrt{2c(a+b)}$

Đặt: $\sqrt{\frac{2c}{b+a}}=t $ thì t thuộc (0;1]

Bài toán đưa về tìm Min của: $f(t)=\frac{2}{\sqrt{t^2+1}}+\frac{t}{\sqrt{2}}+\frac{\sqrt{2}(t^2+1)}{2t+1}, 0

Ta có: $f'(t)=-2t(t^2+1)^{-3/2}+\frac{1}{\sqrt{2}}+2\sqrt{2}(t^2+t-2)(2t+1)^{-2}$ và $f''(t)=2(2t^2-1)(t^2+1)^{-5/2}+18\sqrt{2}(2t+3)^{-3}$

Dễ thấy rằng $f"(t)>0$ với mọi t thuộc (0;1]

Do đó f'(t) có duy nhất nghiệm t=1.

Khảo sát ta được $Minf(t)=f(1)=\frac{5\sqrt{2}}{2}$

Vậy Min P = $\frac{5\sqrt{2}}{2}$. Đẳng thức xảy ra khi a=b=c




#654955 CMR: $\sum \frac{(b+c-a)^2}{(b+c)^2+a^2}...

Đã gửi bởi MrS on 20-09-2016 - 23:37 trong Bất đẳng thức và cực trị

Lời giải của MrS sai ngay từ đây.

Phải xét a+b+c=0 nữa hả anh ơi?  :(




#654961 CMR: $\sum \frac{(b+c-a)^2}{(b+c)^2+a^2}...

Đã gửi bởi MrS on 21-09-2016 - 01:15 trong Bất đẳng thức và cực trị

Để chuẩn hóa $a+b+c=3$ thì em cần phải có $a+b+c>0.$ Nhưng điều này ta có thể giả sử được, em thử nghĩ xem nhé.

Trước em có đọc 1 vài tài liệu thì người ta nói rằng: Nếu thay bộ (a,b,c) bởi (-a,-b,-c) BĐT không đổi thì có thể giả sử a+b+c > 0. Không  biết có đúng không ạ? 




#655028 Topic: [LTDH] Mỗi ngày hai bất đẳng thức.

Đã gửi bởi MrS on 21-09-2016 - 20:25 trong Bất đẳng thức và cực trị

Tiếp theo:

Bài 49: Cho $a,b,c>0$. Chứng minh rằng: $6(a+b+c)(a^2+b^2+c^2)\le 27abc+10(a^2+b^2+c^2)$.

 

Đề bài sai hay sao ý :D: cho $a=10^{-7}, b=c=1 \Rightarrow VP-VT\approx -4< 0$




#655046 Topic: [LTDH] Mỗi ngày hai bất đẳng thức.

Đã gửi bởi MrS on 21-09-2016 - 21:47 trong Bất đẳng thức và cực trị

 

Tiếp theo:

Bài 49: Cho $a,b,c>0$. Chứng minh rằng: $6(a+b+c)(a^2+b^2+c^2)\le 27abc+10(a^2+b^2+c^2)^{\frac{3}{2}}$.

 

 

Bạn xem trong này nhé, nó đúng cả trong trường hợp a,b,c thực.

http://diendantoanho...hức-thuần-nhất/




#655071 Topic: [LTDH] Mỗi ngày hai bất đẳng thức.

Đã gửi bởi MrS on 22-09-2016 - 00:12 trong Bất đẳng thức và cực trị

Bài tập 49:

Do BĐT thuần nhất nên chuẩn hóa $a+b+c=3$

  \[ \to 2{\left( {a + b + c} \right)^2} > {a^2} + {b^2} + {c^2} \ge \frac{{{{\left( {a + b + c} \right)}^2}}}{3}{\rm{ hay }}{a^2} + {b^2} + {c^2} \in \left[ {0;1} \right)\]

\[ \Rightarrow \exists t \in \left[ {0;1} \right]:{a^2} + {b^2} + {c^2} = 3 + 6{t^2} \to ab + bc + ac = 3 - 3{t^2}\]

 

Sao lại có điều này được nhỉ?

Ngoài ra trong bài này chỉ xảy ra đẳng thức khi (x,y,z)=(-1,2,2) hoặc hoán vị. Trong bài này thì không có dấu bằng.

Bài này có thể tham khảo VD4 thầy Dũng viết :) :http://diendantoanho...hức-thuần-nhất/




#655214 Topic: [LTDH] Mỗi ngày hai bất đẳng thức.

Đã gửi bởi MrS on 23-09-2016 - 09:53 trong Bất đẳng thức và cực trị

 

Dưới đây là lời giải bài 49 và bài 50:

 

Lời giải bài 49: Chuẩn hóa: $a^2+b^2+c^2=9$.

 

Ta có: $BDT\iff 2(a+b+c)-abc\le 10$.

 

$VT=2(a+b+c)-abc=2a-abc+2(b+c)=a(2-bc)+2(b+c)$

 

$VT^2\le [a^2+(b+c)^2][(2-bc)^2+4]$.

 

KMTTQ, giả sử: $a\ge b\ge c\implies a^2\ge 3$.

 

Đặt $t=bc\implies t=bc\le \frac{b^2+c^2}{2}=\frac{9-a^2}{2}\le 3$.

 

Nên $VT^2\le (9+2bc)[(2-bc)^2+4]=(9+2t)[(2-t)^2+4]=f(t)\text{   }\forall t\in(0;3]$.

 

Khảo sát $f(t)\implies f(t)\le 100\implies VT\le 10\implies Q.E.D$.

 

Dấu $=$ xảy ra tại $a=b=c$.

 

Không có đẳng thức vì $f(t)=100\Leftrightarrow (t+2)(t^2-3t-14)=0$ :D




#655215 Topic: [LTDH] Mỗi ngày hai bất đẳng thức.

Đã gửi bởi MrS on 23-09-2016 - 10:29 trong Bất đẳng thức và cực trị

Tiếp theo:

Bài 51: Cho các số thực dương $x,y,z$. Tìm GTNN của biểu thức:

 $P=\frac{1}{6\sqrt{xy}+7z+8\sqrt{zx}}-\frac{1}{9\sqrt{x+y+z}}$.

Theo AM-GM: $6\sqrt{xy}+7z+8\sqrt{zx}\leq (x+9y)+7z+2(z+4x)=9(x+y+z)\Rightarrow P\geq \frac{1}{9}(\frac{1}{\sqrt{x+y+z}}-\frac{1}{2})^2-\frac{1}{36}\geq -\frac{1}{36}$

Vậy $MinP=-\frac{1}{36}$, đạt được khi $x=\frac{18}{23}, y=\frac{2}{23}, z=\frac{72}{23}$




#655220 Topic: [LTDH] Mỗi ngày hai bất đẳng thức.

Đã gửi bởi MrS on 23-09-2016 - 11:27 trong Bất đẳng thức và cực trị

Tiếp theo:

Bài 52: Cho các số thực $x,y,z$ thỏa mãn: $x^2+\frac{y^2}{2}+\frac{z^2}{3}=6$. Tìm GTLN của biểu thức:

$P=\frac{(x+y+z)^3-36}{x+y+z}$

Theo Cauchy - Scharzt: $(x^2+\frac{y^2}{2}+\frac{z^2}{3})(1+2+3)\geq (x+y+z)^2\Rightarrow -6\leq x+y+z\leq 6$

Xét $f(t)=\frac{t^3-36}{t}=t^2-\frac{36}{t}, -6\leq t\leq 6$

Tới đây mình tịt :D vì khi t->0- thì f(t)-> +vô cực.

Chắc thêm giả thiết x+y+z>0 thì giải được :D

Không biết đáp án thế nào.




#655532 Topic: [LTDH] Mỗi ngày hai bất đẳng thức.

Đã gửi bởi MrS on 25-09-2016 - 19:37 trong Bất đẳng thức và cực trị

Cảm ơn anh nhiều MrS về bài 49, Thành thật xin lỗi mọi người vì những thiếu sót này. Lần sau mình cố gắng cẩn thận hơn. Mong mọi người thông cảm.

Bài 52  xin đưa ra lời giải như sau:

Theo BDT Cauchy-Schwarz ta có: $(x+y+z)^2\le 6(x^2+\frac{y^2}{2}+\frac{z^2}{3})\le 36$.

$\implies x+y+z\in [-6;6]$.

Đặt $t=x+y+z,t\in [-6;6]$.

Xét hàm số: $f(t)=\frac{t^3-36}{t},t\in [-6;6]$

$\implies f'(t)=2t+\frac{36}{t^2}$.

$f'(t)=0\iff t=-\sqrt[3]{18}$.

$f(-6)=42,f(-\sqrt[3]{18})=\frac{54}{\sqrt[3]{54}},f(6)=30\implies f(t)(Max)=42$.

Dấu $=$ xảy ra tại $x=-1;y=-2;z=-3$.

Vậy $Max(P)=42$.

Bạn quên mất là f(t) bị gián đoạn tại t=0 kìa :)

Khi t->0- thì f(t) -> +vô cực. Tức là khả năng đề bài không đúng :)




#655542 Topic: [LTDH] Mỗi ngày hai bất đẳng thức.

Đã gửi bởi MrS on 25-09-2016 - 20:30 trong Bất đẳng thức và cực trị

Tiếp theo:

Bài 57: Cho $a,b,c$ là các số thực dương thỏa mãn: $a+b+c=3$. Chứng minh rằng:

$18(ab+bc+ca)+a(a-b)^2+b(b-c)^2+c(c-a)^2\le 54$.

BĐT tương đương: $6(ab+bc+ca)(a+b+c)+a(a-b)^2+b(b-c)^2+c(c-a)^2\le 2(a+b+c)^3\Leftrightarrow \sum a^3+2\sum a^2b\geqslant \sum ab^2+6abc$

Giả sử $a\geq b\geq c$ và theo BĐT AM-GM: $\sum a^3+\sum ab^2\geqslant 2\sum a^2b$ và $\sum a^2b\geq 3abc$ 

Mặt khác: $(a-b)(b-c)(a-c)\geq 0\Rightarrow \sum a^2b\geq \sum ab^2$

Từ các BĐT trên suy ra đpcm.

Đẳng thức xảy ra khi a=b=c=1




#655568 Topic: [LTDH] Mỗi ngày hai bất đẳng thức.

Đã gửi bởi MrS on 25-09-2016 - 22:47 trong Bất đẳng thức và cực trị

Hoán vị vòng quanh mà bạn. Nên cần xét thêm trường hợp nữa. 

 \[BDT \Leftrightarrow 2{\left( {a + b + c} \right)^3} \geqslant 6\left( {a + b + c} \right)\left( {ab + bc + ac} \right) + a{\left( {a - b} \right)^2} + b{\left( {b - c} \right)^2} + c{\left( {c - a} \right)^2}\]

\[ \Leftrightarrow \left( {a + b + c} \right)\sum {{{\left( {a - b} \right)}^2}}  \geqslant a{\left( {a - b} \right)^2} + b{\left( {b - c} \right)^2} + c{\left( {c - a} \right)^2}\]

\[ \Leftrightarrow \left( {b + c} \right){\left( {a - b} \right)^2} + \left( {a + c} \right){\left( {b - c} \right)^2} + \left( {a + b} \right){\left( {a - c} \right)^2} \geqslant 0\




#655586 CMR: $\sum \frac{(b+c-a)^2}{(b+c)^2+a^2}...

Đã gửi bởi MrS on 26-09-2016 - 11:52 trong Bất đẳng thức và cực trị

Đúng rồi em. :)

Anh ơi em có vài thắc mắc anh giúp em với :)

"Từ việc thay (a,b,c) bởi (-a,-b,-c) mà BĐT không đổi tại sao lại chỉ cần xét khi a,b,c không âm nhỉ". Nếu đẳng thức xảy là khi có cả số âm, cả không âm thì sao nhỉ? 




#655624 Topic: [LTDH] Mỗi ngày hai bất đẳng thức.

Đã gửi bởi MrS on 26-09-2016 - 17:25 trong Bất đẳng thức và cực trị

Tiếp theo: 

Bài 59: Cho $a,b,c>0$. Chứng minh rằng: $(a^5+\frac{2}{a})(b^5+\frac{2}{b})(c^5+\frac{2}{c})\ge (a+b+c)^3$.

Bài 60: Cho $a,b,c>0$ thỏa mãn: $a^2+b^2+c^2=3$. Chứng minh rằng:

$\frac{1}{4-\sqrt{ab}}+\frac{1}{4-\sqrt{bc}}+\frac{1}{4-\sqrt{ca}}\le 1$

Bài 59: 

Theo BĐT Holder: $(a^5+\frac{2}{a})(b^5+\frac{2}{b})(c^5+\frac{2}{c})\ge (\sqrt[3]{\frac{a^5}{bc}}+\sqrt[3]{\frac{b^5}{ca}}+\sqrt[3]{\frac{c^5}{ab}})^3$

Từ đó chỉ cần chứng minh: $\sqrt[3]{\frac{a^5}{bc}}+\sqrt[3]{\frac{b^5}{ca}}+\sqrt[3]{\frac{c^5}{ab}}\geq a+b+c$

Có nhiều cách CM BĐT này. 

Chuẩn hóa abc=1 khi đó ta cần chứng minh: $\sum a^2\geq \sum a$

Theo AM-GM: $a^2+b^2+c^2\geq \frac{1}{3}(a+b+c)^2, a^2+b^2+c^2\geq3\sqrt[3]{a^2b^2c^2}=3\Rightarrow (a^2+b^2+c^2)^2\geq (a+b+c)^2\Rightarrow \sum a^2\geq \sum a$

Từ đó ta có đpcm. Đẳng thức xảy ra khi a=b=c=1.

 

Bài 60: Theo BĐT AM-GM: $\sum \frac{1}{4-\sqrt{ab}}\leq \sum \frac{1}{4-\sqrt{\frac{a^2+b^2}{2}}}=\sum \frac{1}{4-\sqrt{\frac{3-c^2}{2}}}$

Ta sẽ chứng minh: $\frac{1}{4-\sqrt{\frac{3-x^2}{2}}}\leq \frac{13-x^2}{36}$ (*)

Tới đây chỉ cần đặt: $\sqrt{\frac{3-x^2}{2}}=t \Rightarrow t\in (0,\sqrt{\frac{3}{2}})$

BĐT (*) trở thành: $\frac{1}{4-t}\leq \frac{5+t^2}{18}\Leftrightarrow (t-1)^2(2-t)\geq 0$ luôn đúng do $t\in (0,\sqrt{\frac{3}{2}})$

Từ đó dễ dàng suy ra: $\sum \frac{1}{4-\sqrt{ab}}\leq \sum \frac{13-a^2}{36}=1$

Đẳng thức xảy ra khi a=b=c=1.




#655750 Topic: [LTDH] Mỗi ngày hai bất đẳng thức.

Đã gửi bởi MrS on 27-09-2016 - 17:28 trong Bất đẳng thức và cực trị

Tiếp theo: 

Bài 61: Cho $x,y,z$ là các số thực dương thỏa mãn: $xy\ge 1;z\ge 1$. Tìm GTNN của:

$P=\frac{x}{y+1}+\frac{y}{x+1}+\frac{z^3+2}{3(xy+1)}$.

Bài 61 :

Theo BĐT Cauchy - Schwazt, AM-GM:

$P\geq \frac{x}{y+1}+\frac{y}{x+1}+\frac{1}{xy+1}\geq \frac{(x+y)^2}{2xy+x+y}+\frac{1}{xy+1}\geq \frac{(2\sqrt{xy})^2}{2xy+2\sqrt{xy}}+\frac{1}{xy+1}=\frac{2\sqrt{xy}}{\sqrt{xy}+1}+\frac{1}{xy+1}=\frac{(\sqrt{xy}-1)^3}{2(\sqrt{xy}+1)(xy+1)}+\frac{3}{2}\geq \frac{3}{2}$ do $xy\ge 1;z\ge 1$

Vậy MinP=3/2, dấu bằng xảy ra khi x=y=z=1.